Đến nội dung

Hoang Tung 126 nội dung

Có 1000 mục bởi Hoang Tung 126 (Tìm giới hạn từ 29-04-2020)



Sắp theo                Sắp xếp  

#475064 Topic về Bất đẳng thức, cực trị THCS

Đã gửi bởi Hoang Tung 126 on 03-01-2014 - 20:37 trong Bất đẳng thức và cực trị

Vậy giả sử a+b+c=1 cũng được ạ?

Tùy thôi miễn là tổng của 3 số không âm là được




#474808 Topic về Bất đẳng thức, cực trị THCS

Đã gửi bởi Hoang Tung 126 on 02-01-2014 - 20:12 trong Bất đẳng thức và cực trị

Chỗ này chuẩn hoá nghĩa là sao???

Có nghĩa là đề bài cho 3 ẩn thì mình có thể giả sử được tổng của 3 ẩn ấy 




#473944 Topic về Bất đẳng thức, cực trị THCS

Đã gửi bởi Hoang Tung 126 on 30-12-2013 - 16:38 trong Bất đẳng thức và cực trị

Cho a,b,c>0 .CMR:

 

$\sum \frac{(b+c-a)^{2}}{(b+c)^{2}+a^{2}}\geq \frac{3}{5}$

Bài này có nhiều cách làm Mình xin nêu ra cách ngắn gọn nhất .

Chuẩn hóa :$a+b+c=3$

BĐT $< = > \sum \frac{a(b+c)}{a^2+(b+c)^2}\leq \frac{6}{5}< = > \sum \frac{a(3-a)}{a^2+(3-a)^2}\leq \frac{6}{5}< = > \sum \frac{1}{2a^2-6a+9}\leq \frac{3}{5}$

Mặt khác ta lại có :$\frac{1}{2a^2-6a+9}\leq \frac{2a+3}{25}< = > a^3+a^3+1\geq 3a^2$(Luôn đúng theo AM-GM 3 số)

$= > \sum \frac{1}{2a^2-6a+9}\leq \frac{2\sum a+9}{25}=\frac{2.3+9}{25}=\frac{3}{5}$(đpcm)

 Dấu = xảy ra khi $a=b=c=1$




#537456 Bộ sưu tập TC Toán học và Tuổi trẻ từ năm 1994 đến nay

Đã gửi bởi Hoang Tung 126 on 12-12-2014 - 17:58 trong Toán học & Tuổi trẻ

Sao tải mấy bản này toàn lỗi PDF vậy




#482913 $\boxed{\text{Chuyên Đề}}$ Bất đẳng thức - Cực trị

Đã gửi bởi Hoang Tung 126 on 13-02-2014 - 18:25 trong Bất đẳng thức và cực trị

52) Cho $\left\{\begin{matrix}a;b;c>0 & & \\ a+b+c=4 & & \end{matrix}\right.$. Cmr: $4\leq \sum \sqrt{a+b}\leq 2\sqrt{6}$

 

53) Cho $a;b;c$ là độ dài 3 cạnh một tam giác, Cmr: $\sum \sqrt{a+b-c}\leq \sum \sqrt{a}$

 

54) Cho $a;b;c$ là độ dài 3 cạnh một tam giác có chu vi bằng $2$. Cmr: $\sum a^2<2(1-abc)$

 

55) Xác định dạng tam giác $ABC$ có chu vi bằng $1$ biết $A=\sum \frac{a}{1-2a}$ đạt $min$.

 

56) Cho $a;b;c>0$ thỏa $a^2+b^2+c^2=1$. Cmr: $\sum \frac{a^2}{1+b-a}\geq 1$

 

57) Cho $a;b;c$ là độ dài 3 cạnh một tam giác. Cmr: $\sum \sqrt[3]{a+b-c}\leq \sum \sqrt[3]{a}$

Bài 54:Theo bdt Tam giác có:$1=a+b+c> a+a=2a= > 1-2a> 0$.Tương tự $1-2b> 0,1-2c> 0$

Nhân theo vế $= > (1-2a)(1-2b)(1-2c)> 0< = > 4(\sum ab)> 2\sum a+8abc-1=1+8abc= > 2(\sum a)^2> 2\sum a^2+8abc+1= > \sum a^2+4abc< 1$(DPCM)




#482912 $\boxed{\text{Chuyên Đề}}$ Bất đẳng thức - Cực trị

Đã gửi bởi Hoang Tung 126 on 13-02-2014 - 18:20 trong Bất đẳng thức và cực trị

52) Cho $\left\{\begin{matrix}a;b;c>0 & & \\ a+b+c=4 & & \end{matrix}\right.$. Cmr: $4\leq \sum \sqrt{a+b}\leq 2\sqrt{6}$

 

53) Cho $a;b;c$ là độ dài 3 cạnh một tam giác, Cmr: $\sum \sqrt{a+b-c}\leq \sum \sqrt{a}$

 

54) Cho $a;b;c$ là độ dài 3 cạnh một tam giác có chu vi bằng $2$. Cmr: $\sum a^2<2(1-abc)$

 

55) Xác định dạng tam giác $ABC$ có chu vi bằng $1$ biết $A=\sum \frac{a}{1-2a}$ đạt $min$.

 

56) Cho $a;b;c>0$ thỏa $a^2+b^2+c^2=1$. Cmr: $\sum \frac{a^2}{1+b-a}\geq 1$

 

57) Cho $a;b;c$ là độ dài 3 cạnh một tam giác. Cmr: $\sum \sqrt[3]{a+b-c}\leq \sum \sqrt[3]{a}$

Bài 53:Theo Bunhiacopxki có:$\sqrt{a+b-c}+\sqrt{b+c-a}\leq \sqrt{2(a+b-c+b+c-a)}=\sqrt{4b}=2\sqrt{b}$

                                             $\sqrt{a+b-c}+\sqrt{a+c-b}\leq 2\sqrt{a}$

                                             $\sqrt{b+c-a}+\sqrt{a+c-b}\leq 2\sqrt{b}$

Cộng theo vế các pt $= > 2\sum \sqrt{a+c-b}\leq 2\sum \sqrt{a}= > \sum \sqrt{a+c-b}\leq \sum \sqrt{a}$(ĐPCM)




#482914 $\boxed{\text{Chuyên Đề}}$ Bất đẳng thức - Cực trị

Đã gửi bởi Hoang Tung 126 on 13-02-2014 - 18:27 trong Bất đẳng thức và cực trị

52) Cho $\left\{\begin{matrix}a;b;c>0 & & \\ a+b+c=4 & & \end{matrix}\right.$. Cmr: $4\leq \sum \sqrt{a+b}\leq 2\sqrt{6}$

 

53) Cho $a;b;c$ là độ dài 3 cạnh một tam giác, Cmr: $\sum \sqrt{a+b-c}\leq \sum \sqrt{a}$

 

54) Cho $a;b;c$ là độ dài 3 cạnh một tam giác có chu vi bằng $2$. Cmr: $\sum a^2<2(1-abc)$

 

55) Xác định dạng tam giác $ABC$ có chu vi bằng $1$ biết $A=\sum \frac{a}{1-2a}$ đạt $min$.

 

56) Cho $a;b;c>0$ thỏa $a^2+b^2+c^2=1$. Cmr: $\sum \frac{a^2}{1+b-a}\geq 1$

 

57) Cho $a;b;c$ là độ dài 3 cạnh một tam giác. Cmr: $\sum \sqrt[3]{a+b-c}\leq \sum \sqrt[3]{a}$

Bài 55:Ta có:$\sum \frac{a}{1-2a}=\sum \frac{a^2}{a-2a^2}\geq \frac{(\sum a)^2}{\sum a-2\sum a^2}=\frac{1}{1-2\sum a^2}\geq \frac{1}{1-2.\frac{(\sum a)^2}{3}}=\frac{1}{1-\frac{2}{3}}=3$




#482724 $\boxed{\text{Chuyên Đề}}$ Bất đẳng thức - Cực trị

Đã gửi bởi Hoang Tung 126 on 12-02-2014 - 16:48 trong Bất đẳng thức và cực trị

Bài 44:Theo Bunhiacopxki co:$\sum \sqrt{2x^2+xy+y^2}=\sum \sqrt{x^2+\frac{3}{4}(x+y)^2+\frac{1}{4}(x-y)^2}\geq \sum \sqrt{x^2+\frac{3(x+y)^2}{4}}=\frac{1}{2}\sum \sqrt{4x^2+3(x+y)^2}=\frac{1}{2}\sum \sqrt{(2x)^2+(x+y)^2+(x+y)^2+(x+y)^2}\geq \frac{1}{2}\sum \frac{2x+x+y+x+y+x+y}{4}=\frac{1}{2}.\sum \frac{5x+3y}{4}=\sum x=1$




#483035 $\boxed{\text{Chuyên Đề}}$ Bất đẳng thức - Cực trị

Đã gửi bởi Hoang Tung 126 on 14-02-2014 - 08:32 trong Bất đẳng thức và cực trị

58) Cho $x\in (0;1)$. Tìm Min $A=\frac{4x^2+1}{x^2(1-x)}$

 

59) Cho $x\in (0;3)$. Tìm Max $B=(5x^2-14x-3)(x-3)$

 

60) Cho $a;b;c$ là độ dài 3 cạnh một tam giác. Cmr: $\sum \frac{a}{\sqrt{b+c-a}}\geq \sum \sqrt{a}$

 

61) Cho $a;b;c$ là độ dài 3 cạnh một tam giác. Cmr: $|\sum \frac{a-b}{a+b}|<\frac{1}{8}$

 

62) Cho $x\in [0;1]$. Cmr: $\sqrt[4]{1-x^2}+\sqrt[4]{1-x}+\sqrt[4]{1+x}\leq 3$

 

63) Cho $a;b;c>0$. Cmr: $\frac{1}{a^3}+\frac{a^3}{b^3}+b^3\geq \frac{1}{a}+\frac{a}{b}+b$

 

64)

a) Cho $x\in [-1;1]$. Cmr: $|4x^3-3x|\leq 1$

b) Cho $\left\{\begin{matrix}a_1;a_2;...;a_n\in [-1;1] & & \\ a_1^3+a_2^3+...+a_n^3=0 & & \end{matrix}\right.$. Cmr: $a_1+a_2+...+a_n\leq \frac{n}{3}$

 

65) Cho $x;y>0$ thỏa $x^2+y^2=1$. Cmr: $xy+Max(x;y)\leq \frac{3\sqrt{3}}{4}$ (Giải thích: Nếu $x>y$ thì $Max(x;y)=x$ và tương tự)

 

 

P/s: Anh Daicagiangho1998 học KHTN nên cứ từ từ mà làm thôi, chứ có đề phát đã ăn hết sạch luôn vậy :D

Bài 65:Bài này khá hay:Giả sử $x=Max(x,y)$

 Theo AM-GM có:$xy\leq \frac{x^2+3y^2}{2\sqrt{3}},x\leq \frac{4x^2+3}{4\sqrt{3}}$

$= > xy+max(x,y)=x+y\leq \frac{2(x^2+3y^2)+(4x^2+3)}{4\sqrt{3}}=\frac{6(x^2+y^2)+3}{4\sqrt{3}}=\frac{3\sqrt{3}}{4}$

Dấu = xảy ra tại $x=\frac{\sqrt{3}}{2},y=\frac{1}{2}$




#495292 $\boxed{\text{Chuyên Đề}}$ Bất đẳng thức - Cực trị

Đã gửi bởi Hoang Tung 126 on 26-04-2014 - 19:35 trong Bất đẳng thức và cực trị

Bài 153: 
Cho $a,b,c\geq 0$ thỏa $a+b+c=3$. Tìm GTLN của: 
$S=(a^2-ab+b^2)(b^2-bc+c^2)(c^2-ca+a^2)$

Bài này chưa ai giải thì xơi vậy...

Gỉa sử $a\geq b\geq c= > b^2-bc+c^2=b^2+c(c-b)\leq b^2,c^2-ac+a^2=a^2+c(c-a)\leq a^2$

$= > S\leq a^2b^2(a^2-ab+b^2)=\frac{4}{9}.\frac{3ab}{2}.\frac{3ab}{2}(a^2-ab+b^2)\leq \frac{4}{9}.\frac{(\frac{3ab}{2}+\frac{3ab}{2}+a^2-ab+b^2)^3}{27}=\frac{4(a+b)^6}{9.27}\leq \frac{4(a+b+c)^6}{9.27}=\frac{4.3^6}{9.27}=12= > S\leq 12$

Dấu = xảy ra khi $c=0,a=1,b=2$

 

Bài này em đưa link giải rồi Hì :)

 


 

Ở đây

 

 




#491603 $\boxed{\text{Chuyên Đề}}$ Bất đẳng thức - Cực trị

Đã gửi bởi Hoang Tung 126 on 09-04-2014 - 11:09 trong Bất đẳng thức và cực trị

Bài 168: Cho $a,b,c$ thỏa mãn:$a^2+b^2+c^2=1$.

Tìm GTLN của biểu thức: $A=(a-b)(b-c)(c-a)(a+b+c)$

 

Bài 169: Cho các số thực $a,b,c$. Chứng minh rằng:

$\sum \sqrt{\frac{a(b+c)}{a^2+bc}}\leq \sqrt{\sum \sqrt{a}.\sum \frac{1}{\sqrt{a}}}$

Bài 169:Ta có:$(\sum \sqrt{a})(\sum \frac{1}{\sqrt{a}})\geq 3\sqrt[3]{\sqrt{abc}}.3\sqrt[3]{\frac{1}{\sqrt{abc}}}=9= > \sqrt{(\sum \sqrt{a})(\sum \frac{1}{\sqrt{a}})}\geq 3$

Mà $\sum \sqrt{\frac{a(b+c)}{a^2+bc}}\leq \sqrt{3(\sum \frac{a(b+c)}{a^2+bc})}\leq 3< = = > \sum \frac{a(b+c)}{a^2+bc}\leq 3< = > \sum (1-\frac{a(b+c)}{a^2+bc})\geq 0< = > \frac{(a-b)(a-c)}{a^2+bc}+\frac{(b-c)(b-a)}{b^2+ac}+\frac{(c-a)(c-b)}{c^2+ab}\geq 0$

Nhưng bđt này luôn đúng vì đây là Schur mở rộng 




#482722 $\boxed{\text{Chuyên Đề}}$ Bất đẳng thức - Cực trị

Đã gửi bởi Hoang Tung 126 on 12-02-2014 - 16:40 trong Bất đẳng thức và cực trị

Bài 43:Ta có:$\sum \sqrt{x^2+xy+y^2}=\sum \sqrt{\frac{3(x+y)^2}{4}+\frac{(x-y)^2}{4}}\geq \sum \sqrt{\frac{3(x+y)^2}{4}}=\sum \frac{\sqrt{3}(x+y)}{2}=\sqrt{3}(\sum x)=\sqrt{3}.1=\sqrt{3}$

 Đẳng thức xảy ra tại $x=y=z=\frac{1}{3}$




#483034 $\boxed{\text{Chuyên Đề}}$ Bất đẳng thức - Cực trị

Đã gửi bởi Hoang Tung 126 on 14-02-2014 - 08:28 trong Bất đẳng thức và cực trị

58) Cho $x\in (0;1)$. Tìm Min $A=\frac{4x^2+1}{x^2(1-x)}$

 

59) Cho $x\in (0;3)$. Tìm Max $B=(5x^2-14x-3)(x-3)$

 

60) Cho $a;b;c$ là độ dài 3 cạnh một tam giác. Cmr: $\sum \frac{a}{\sqrt{b+c-a}}\geq \sum \sqrt{a}$

 

61) Cho $a;b;c$ là độ dài 3 cạnh một tam giác. Cmr: $|\sum \frac{a-b}{a+b}|<\frac{1}{8}$

 

62) Cho $x\in [0;1]$. Cmr: $\sqrt[4]{1-x^2}+\sqrt[4]{1-x}+\sqrt[4]{1+x}\leq 3$

 

63) Cho $a;b;c>0$. Cmr: $\frac{1}{a^3}+\frac{a^3}{b^3}+b^3\geq \frac{1}{a}+\frac{a}{b}+b$

 

64)

a) Cho $x\in [-1;1]$. Cmr: $|4x^3-3x|\leq 1$

b) Cho $\left\{\begin{matrix}a_1;a_2;...;a_n\in [-1;1] & & \\ a_1^3+a_2^3+...+a_n^3=0 & & \end{matrix}\right.$. Cmr: $a_1+a_2+...+a_n\leq \frac{n}{3}$

 

65) Cho $x;y>0$ thỏa $x^2+y^2=1$. Cmr: $xy+Max(x;y)\leq \frac{3\sqrt{3}}{4}$ (Giải thích: Nếu $x>y$ thì $Max(x;y)=x$ và tương tự)

 

 

P/s: Anh Daicagiangho1998 học KHTN nên cứ từ từ mà làm thôi, chứ có đề phát đã ăn hết sạch luôn vậy :D

Bài 64:Ta có:$\left | 4x^3-3x \right |\leq 1< = > -1\leq 4x^3-3x\leq 1< = > 4x^3-3x+1\geq 0,1+3x-4x^3\geq 0< = > (2x-1)^2(x+1)\geq 0,(1-x)(4x^2-4x+1)\geq 0$(Luôn đúng theo giả thiết)

 

 

 



#483033 $\boxed{\text{Chuyên Đề}}$ Bất đẳng thức - Cực trị

Đã gửi bởi Hoang Tung 126 on 14-02-2014 - 08:18 trong Bất đẳng thức và cực trị

58) Cho $x\in (0;1)$. Tìm Min $A=\frac{4x^2+1}{x^2(1-x)}$

 

59) Cho $x\in (0;3)$. Tìm Max $B=(5x^2-14x-3)(x-3)$

 

60) Cho $a;b;c$ là độ dài 3 cạnh một tam giác. Cmr: $\sum \frac{a}{\sqrt{b+c-a}}\geq \sum \sqrt{a}$

 

61) Cho $a;b;c$ là độ dài 3 cạnh một tam giác. Cmr: $|\sum \frac{a-b}{a+b}|<\frac{1}{8}$

 

62) Cho $x\in [0;1]$. Cmr: $\sqrt[4]{1-x^2}+\sqrt[4]{1-x}+\sqrt[4]{1+x}\leq 3$

 

63) Cho $a;b;c>0$. Cmr: $\frac{1}{a^3}+\frac{a^3}{b^3}+b^3\geq \frac{1}{a}+\frac{a}{b}+b$

 

64)

a) Cho $x\in [-1;1]$. Cmr: $|4x^3-3x|\leq 1$

b) Cho $\left\{\begin{matrix}a_1;a_2;...;a_n\in [-1;1] & & \\ a_1^3+a_2^3+...+a_n^3=0 & & \end{matrix}\right.$. Cmr: $a_1+a_2+...+a_n\leq \frac{n}{3}$

 

65) Cho $x;y>0$ thỏa $x^2+y^2=1$. Cmr: $xy+Max(x;y)\leq \frac{3\sqrt{3}}{4}$ (Giải thích: Nếu $x>y$ thì $Max(x;y)=x$ và tương tự)

 

 

P/s: Anh Daicagiangho1998 học KHTN nên cứ từ từ mà làm thôi, chứ có đề phát đã ăn hết sạch luôn vậy :D

Bài 62:Theo AM-GM 4 số có:$\sqrt[4]{1-x^2}=\sqrt[4]{(1-x)(1+x).1.1}\leq \frac{1-x+1+x+1+1}{4}=1$

                                             $\sqrt[4]{(1-x).1.1.1}\leq \frac{1-x+1+1+1}{4}=\frac{4-x}{4}$

                                             $\sqrt[4]{(1+x).1.1.1}\leq \frac{1+x+1+1+1}{4}=\frac{4+x}{4}$

Cộng theo vế các bdt $= > \sqrt[4]{1-x^2}+\sqrt[4]{1+x}+\sqrt[4]{1-x}\leq \frac{4-x+4+x}{4}+1=3$




#482720 $\boxed{\text{Chuyên Đề}}$ Bất đẳng thức - Cực trị

Đã gửi bởi Hoang Tung 126 on 12-02-2014 - 16:36 trong Bất đẳng thức và cực trị

41) Cho $x;y;z>0$. Cmr: $\sum\frac{2\sqrt{x}}{x^3+y^2}\leq \sum \frac{1}{x^2}$

 

42) Cho $a;b;c>0$. Cmr: $\sum \frac{1}{a^2+bc}\leq \frac{a+b+c}{2abc}$

 

43) Cho $\left\{\begin{matrix}x;y;z>0 & & \\ x+y+z=1 & & \end{matrix}\right.$. Cmr: $\sum \sqrt{x^2+xy+y^2}\geq \sqrt{3}$

 

44) Cho $\left\{\begin{matrix}x;y;z>0 & & \\ x+y+z=1 & & \end{matrix}\right.$. Cmr: $\sum \sqrt{2x^2+xy+y^2}\geq \sqrt{5}$

 

45) Cho $a;b>0$. Tìm Max $Q=a^3+b^3$ biết $a+b=a^2-ab+b^2$ 

 

46) Cho $a;b;c>0$ thỏa $a^2+b^2+c^2=1$. Cmr: $\sum \frac{1}{a^2+b^2}\leq \frac{\sum a^3}{2abc}+3$

Bài 41:Theo AM-GM có:$\sum \frac{2\sqrt{x}}{x^3+y^2}\leq \sum \frac{2\sqrt{x}}{2\sqrt{x^3y^2}}=\sum \frac{1}{xy}\leq \sum \frac{1}{x^2}$

Đẳng thức xảy ra tại $x=y=z=1$




#482127 $\boxed{\text{Chuyên Đề}}$ Bất đẳng thức - Cực trị

Đã gửi bởi Hoang Tung 126 on 09-02-2014 - 09:34 trong Bất đẳng thức và cực trị

Bài 29:Áp dụng bđt $x^4+y^4\geq \frac{(x+y)^4}{8}$

$= > \sum \sqrt[4]{a^4+b^4}\geq \sum \sqrt[4]{\frac{(a+b)^4}{8}}=\sum \frac{a+b}{\sqrt[4]{8}}=\sqrt[4]{2}\sum a$




#482126 $\boxed{\text{Chuyên Đề}}$ Bất đẳng thức - Cực trị

Đã gửi bởi Hoang Tung 126 on 09-02-2014 - 09:33 trong Bất đẳng thức và cực trị

Bài 28:Ta có:$\sum \sqrt[3]{a^3+b^3}\geq \sum \frac{a+b}{\sqrt[3]{4}}=\sqrt[3]{2}\sum a$




#482012 $\boxed{\text{Chuyên Đề}}$ Bất đẳng thức - Cực trị

Đã gửi bởi Hoang Tung 126 on 08-02-2014 - 19:14 trong Bất đẳng thức và cực trị

23) Cho $\left\{\begin{matrix}a;b;c>0 & & \\ a+b+c=1 & & \end{matrix}\right.$. Cmr: $\sum \frac{1}{a+bc}\geq \frac{27}{4}$

 

24) Cho $\left\{\begin{matrix}a;b;c>0 & & \\ a^3c+b^3a+c^3b=abc & & \end{matrix}\right.$. Cmr: $\sum \frac{b}{a^2+ab}\geq \frac{9}{2}$

 

25) Cho $a;b;c>0$. Cmr: $\sum \frac{ab}{a+b+2c}\leq \frac{\sum a}{4}$

 

26) Cho $a;b;c>0$. Cmr: $\sum \frac{ab}{a+3b+2c}\leq \frac{\sum a}{6}$

Bài 26:Ta có:$\sum \frac{ab}{a+3b+2c}=\sum \frac{ab}{(c+b)+(c+a)+2b}\leq \frac{1}{9}(\sum \frac{ab}{c+b}+\sum \frac{ab}{a+c}+\sum \frac{ab}{2b})=\frac{1}{9}(\sum \frac{ab}{c+b}+\sum \frac{ac}{c+b}+\sum \frac{a}{2})=\frac{\sum a}{6}$




#482011 $\boxed{\text{Chuyên Đề}}$ Bất đẳng thức - Cực trị

Đã gửi bởi Hoang Tung 126 on 08-02-2014 - 19:11 trong Bất đẳng thức và cực trị

23) Cho $\left\{\begin{matrix}a;b;c>0 & & \\ a+b+c=1 & & \end{matrix}\right.$. Cmr: $\sum \frac{1}{a+bc}\geq \frac{27}{4}$

 

24) Cho $\left\{\begin{matrix}a;b;c>0 & & \\ a^3c+b^3a+c^3b=abc & & \end{matrix}\right.$. Cmr: $\sum \frac{b}{a^2+ab}\geq \frac{9}{2}$

 

25) Cho $a;b;c>0$. Cmr: $\sum \frac{ab}{a+b+2c}\leq \frac{\sum a}{4}$

 

26) Cho $a;b;c>0$. Cmr: $\sum \frac{ab}{a+3b+2c}\leq \frac{\sum a}{6}$

Bài 24:Ta có:$\sum \frac{ab}{a+b+2c}=\sum \frac{ab}{(a+c)+(a+c)}\leq \frac{1}{4}(\sum \frac{ab}{a+c}+\sum \frac{ab}{a+c})=\frac{1}{4}(\sum \frac{ab}{a+c}+\sum \frac{bc}{a+c})=\frac{1}{4}(\sum a)$




#482128 $\boxed{\text{Chuyên Đề}}$ Bất đẳng thức - Cực trị

Đã gửi bởi Hoang Tung 126 on 09-02-2014 - 09:36 trong Bất đẳng thức và cực trị

Bài 30:Áp dụng bđt $x^5+y^5\geq \frac{(x+y)^5}{16}$

$= > \sum \sqrt[5]{a^5+b^5}\geq \sum \sqrt[5]{\frac{(a+b)^5}{16}}=\sum \frac{a+b}{\sqrt[5]{16}}=\sqrt[5]{2}\sum a$




#482178 $\boxed{\text{Chuyên Đề}}$ Bất đẳng thức - Cực trị

Đã gửi bởi Hoang Tung 126 on 09-02-2014 - 13:03 trong Bất đẳng thức và cực trị

Giờ sẽ là BĐT và Cực Trị nhé.

1) (BĐT Schur)

Cmr: $\sum a^3+3abc\geq \sum ab(a+b)$

2) Tìm Min; Max của $A=xy$ biết $x;y$ nguyên dương và $x+y=2005$.

3) Tìm Min $A=|11^m-5^n|$ với $m;n$ nguyên dương.

4) Cho $x;y;z;t$ dương thỏa $x+y+z+t=2$

Tìm Min $A=\frac{(x+y+z)(x+y)}{xyzt}$

5) Cho $a;b;c>0$. Cmr: $\sum \sqrt{\frac{a}{b+c}}\geq 2$

6) Cho $a;b;c>0$. Cmr: $\sum \sqrt{\frac{a(b+c)}{a^2+bc}}$

Bài 1:BĐT $< = > x(x-y)(x-z)+y(y-z)(y-x)+z(z-x)(z-y)\geq 0$

Không mất tổng quát giả sử $x\leq y\leq z= > z(z-x)(z-y)\geq 0$

Do đó ta cần CM :$y(y-z)(y-x)+x(x-z)(x-y)\geq 0< = > (x-y)(x^2-xz-y^2+yz)\geq 0< = > (x-y)((x-y)(y+x)-z(x-y))\geq 0< = > (x-y)^2(x+y-z)\geq 0$(Luôn đúng do $x\geq y\geq z$)




#482238 $\boxed{\text{Chuyên Đề}}$ Bất đẳng thức - Cực trị

Đã gửi bởi Hoang Tung 126 on 09-02-2014 - 16:02 trong Bất đẳng thức và cực trị

Bài 11:Ta có:$A=2\sum ab+\sum \frac{1}{ab}=2\sum ab+\frac{\sum a}{abc}=2\sum ab+\frac{3}{abc}=\sum ab+\sum ab+\frac{3}{abc}\geq 3\sqrt[3]{\frac{(\sum ab)^2}{abc}}\geq 3\sqrt[3]{\frac{3abc(\sum a).3}{abc}}=3\sqrt[3]{9(\sum a)}=3\sqrt[3]{3.9}=9$

Dấu = xảy ra tại a=b=c=1




#482236 $\boxed{\text{Chuyên Đề}}$ Bất đẳng thức - Cực trị

Đã gửi bởi Hoang Tung 126 on 09-02-2014 - 16:00 trong Bất đẳng thức và cực trị

Bài 7: BĐT $< = > (\sum a)(\sum \frac{1}{a})-3\geq 3\sqrt[3]{\frac{3(a+b+c)(a+b)(b+c)(c+a)}{(ab+bc+ac)^2}}< = > \sum \frac{a+b}{c}\geq 3\sqrt[3]{\frac{3(a+b+c)(a+b)(b+c)(c+a)}{(ab+bc+ac)^2}}$

Mặt khác theo AM-GM 3 số có:$\sum \frac{a+b}{c}\geq 3\sqrt[3]{\frac{(a+b)(b+c)(c+a)}{abc}}$

 Do đó ta cần CM:$3\sqrt[3]{\frac{(a+b)(b+c)(c+a)}{abc}}\geq 3\sqrt[3]{\frac{3(a+b+c)(a+b)(b+c)(c+a)}{(ab+bc+ac)^2}}< = > (ab+bc+ac)^2\geq 3abc(a+b+c)< = > (ab-ac)^2+(bc-ba)^2+(ca-cb)^2\geq 0$

(Luôn đúng)

 Do đó ta có ĐPCM. Dấu = xảy ra tại $a=b=c$




#482226 $\boxed{\text{Chuyên Đề}}$ Bất đẳng thức - Cực trị

Đã gửi bởi Hoang Tung 126 on 09-02-2014 - 15:33 trong Bất đẳng thức và cực trị

Bài 14:Theo AM-GM có:$P=\frac{a}{\sqrt{a(b+2c)}}+\frac{b}{\sqrt{b(c+2a)}}+\frac{c}{\sqrt{c(a+b+c)}}+\frac{c}{\sqrt{c(a+b+c)}}\geq \frac{a}{\frac{a+b+2c}{2}}+\frac{b}{\frac{b+c+2a}{2}}+\frac{c}{\frac{a+b+2c}{2}}+\frac{c}{\frac{a+b+2c}{2}}=2(\frac{a}{a+b+2c}+\frac{b}{b+c+2a}+\frac{c}{a+b+2c}+\frac{c}{a+b+2c})=2(\frac{a^2}{ab+a^2+2ac}+\frac{b^2}{b^2+bc+2ab}+\frac{c^2}{ac+bc+2c^2}+\frac{c^2}{ac+bc+2c^2})\geq 2.\frac{(a+b+2c)^2}{a^2+b^2+4c^2+3ab+3ac+3bc}=2.\frac{a^2+b^2+4c^2+2ab+4ac+4bc}{a^2+b^2+4c^2+3ab+3ac+3bc}\geq 2< = > ac+bc\geq ab$




#482721 $\boxed{\text{Chuyên Đề}}$ Bất đẳng thức - Cực trị

Đã gửi bởi Hoang Tung 126 on 12-02-2014 - 16:37 trong Bất đẳng thức và cực trị

Bài 42:Theo AM-GM có:$\sum \frac{1}{a^2+bc}\leq \sum \frac{1}{2a\sqrt{bc}}=\frac{\sum \sqrt{bc}}{2abc}\leq \frac{\sum a}{2abc}$

Dấu = xảy ra tại a=b=c=1